Liczba e - granica ciągu czy funkcji?

Zbiór wzorów, definicji i najczęściej poruszanych problemów z Analizy.
Awatar użytkownika
Dasio11
Moderator
Moderator
Posty: 10211
Rejestracja: 21 kwie 2009, o 19:04
Płeć: Mężczyzna
Lokalizacja: Wrocław
Podziękował: 40 razy
Pomógł: 2359 razy

Liczba e - granica ciągu czy funkcji?

Post autor: Dasio11 »

Liczbę Eulera \(\displaystyle{ e}\) przyjęło się definiować między innymi na dwa równoważne sposoby:

\(\displaystyle{ \mbox{I}. \ e = \lim_{n \to \infty} \left( 1+\frac{1}{n} \right)^n \\ \\ \\
\mbox{II}. \ e = \lim_{x \to 0} (1+x)^{\frac{1}{x}}}\)


Jak można zauważyć, pierwsza równość wynika z drugiej - wystarczy podstawić \(\displaystyle{ x=\frac{1}{n}.}\) Wydawać by się mogło, że druga definicja jest przeto bardziej wygodna, bo można za jej pomocą liczyć ogólniejsze granice, na przykład

\(\displaystyle{ \lim_{n \to \infty} \left( 1+ \ctg \left( \frac{\pi}{2} -\frac{1}{n} \right) \right)^{\tg \left( \frac{\pi}{2} - \frac{1}{n} \right)} = e.}\)

Jak to w matematyce bywa, korzyści nie są jednak darmowe, a w rzeczywistości obie definicje wymagają takiej samej ilości wysiłku, aby można było z nich korzystać w sposób jednakowy. Przejdźmy więc do konkretów.


Fakt 1. Ciąg \(\displaystyle{ e_n = \left(1+\frac{1}{n} \right)^n}\) jest rosnący i ograniczony, więc jest zbieżny.

Dowód.

Korzystając ze wzoru dwumianowego Newtona

\(\displaystyle{ (a+b)^n = \sum_{j=0}^n {n \choose j} a^j b^{n-j}}\)

rozpiszmy wyrażenia \(\displaystyle{ e_n}\) oraz \(\displaystyle{ e_{n+1}}\) w taki sposób:

\(\displaystyle{ \left(1+\frac{1}{n} \right)^n ={n \choose 0} \cdot 1+ {n \choose 1} \cdot \frac{1}{n} + {n \choose 2} \cdot \frac{1}{n^2} + {n \choose 3} \cdot \frac{1}{n^3} + \ldots + {n \choose n} \cdot \frac{1}{n^n} = \\ \\ \\
=1 + n \cdot \frac{1}{n} + \frac{n(n-1)}{2} \cdot \frac{1}{n^2} + \frac{n(n-1)(n-2)}{3!} \cdot \frac{1}{n^3} + \ldots + { \frac{n(n-1)(n-2) \cdots (n-(n-1))}{n!} \cdot \frac{1}{n^n} } = \\ \\
=1+1+\frac{1}{2!} \cdot \frac{n(n-1)}{n^2} + \frac{1}{3!} \cdot \frac{n(n-1)(n-2)}{n^3} + \ldots + \frac{1}{n!} \cdot \frac{n(n-1)(n-2) \cdots (n-(n-1))}{n^n} = \\ \\
=1+1+\frac{1}{2!} \cdot \left(1-\frac{1}{n} \right) + \frac{1}{3!} \cdot \left(1-\frac{1}{n} \right) \left(1-\frac{2}{n} \right) + \ldots + \frac{1}{n!} \cdot \left(1-\frac{1}{n} \right) \left(1-\frac{2}{n} \right) \left(1-\frac{3}{n} \right) \cdots \left(1-\frac{n-1}{n} \right),\\ \\ \\ \\
\left(1+\frac{1}{n+1} \right)^{n+1} = 1+1+\frac{1}{2!} \cdot \left(1-\frac{1}{n+1} \right) + \frac{1}{3!} \cdot \left(1-\frac{1}{n+1} \right) \left(1-\frac{2}{n+1} \right) + \\ \\ \\
+ \ldots + \frac{1}{n!} \cdot \left(1-\frac{1}{n+1} \right) \left(1-\frac{2}{n+1} \right) \left(1-\frac{3}{n+1} \right) \cdots \left(1-\frac{n-1}{n+1} \right) + \\ \\ \\
+\frac{1}{(n+1)!} \cdot \left(1-\frac{1}{n+1} \right) \left(1-\frac{2}{n+1} \right) \left(1-\frac{3}{n+1} \right) \cdots \left(1-\frac{n-1}{n+1} \right) \left(1-\frac{n}{n+1} \right).}\)



Można zauważyć, że dla każdego \(\displaystyle{ k=2, 3, 4, \ldots, n}\) współczynnik

\(\displaystyle{ \left(1-\frac{1}{n} \right) \left(1-\frac{2}{n} \right) \left(1-\frac{3}{n} \right) \cdots \left(1-\frac{k-1}{n} \right)}\)

przy ułamku \(\displaystyle{ \frac{1}{k!}}\) w wyrażeniu \(\displaystyle{ e_n}\) jest mniejszy, niż odpowiedni współczynnik

\(\displaystyle{ \left(1-\frac{1}{n+1} \right) \left(1-\frac{2}{n+1} \right) \left(1-\frac{3}{n+1} \right) \cdots \left(1-\frac{k-1}{n+1} \right)}\)

w wyrażeniu \(\displaystyle{ e_{n+1}.}\) Ponadto, w \(\displaystyle{ e_{n+1}}\) mamy dodatkowy składnik

\(\displaystyle{ \frac{1}{(n+1)!} \cdot \left(1-\frac{1}{n+1} \right) \left(1-\frac{2}{n+1} \right) \left(1-\frac{3}{n+1} \right) \cdots \left(1-\frac{n-1}{n+1} \right) \left(1-\frac{n}{n+1} \right).}\)

Wnioskujemy stąd, że \(\displaystyle{ e_n < e_{n+1}.}\)

Z rozpisania wynika też, że

\(\displaystyle{ e_n <1+1+\frac{1}{2!} + \frac{1}{3!} + \frac{1}{4!} + \ldots + \frac{1}{n!} = \\ \\
=1+1+\frac{1}{2} + \frac{1}{2 \cdot 3} + \frac{1}{2 \cdot 3 \cdot 4} + \ldots + \frac{1}{2 \cdot 3 \cdot 4 \cdots (n-1) \cdot n} \le \\ \\
\le 1+1+\frac{1}{2} + \frac{1}{2^2} + \frac{1}{2^3} + \ldots + \frac{1}{2^{n-1}} = 1+ \frac{1- \left( \frac{1}{2} \right)^n}{1-\frac{1}{2}} < 1+\frac{1}{\frac{1}{2}} = 3.}\)


\(\displaystyle{ \left( e_n \right)}\) jest więc rosnący i ograniczony z góry przez \(\displaystyle{ 3,}\) więc jest zbieżny. \(\displaystyle{ \blacktriangledown}\)


Teraz, skoro wiemy już, że ciąg ten jest zbieżny, możemy śmiało przyjąć definicję \(\displaystyle{ \mbox{I}}\):

\(\displaystyle{ e=\lim_{n \to \infty} \left(1+\frac{1}{n} \right)^n.}\)


Fakt 2. Funkcja \(\displaystyle{ g: \RR_+ \to \RR}\) dana wzorem

\(\displaystyle{ g(x) = \left(1+\frac{1}{x} \right)^x}\)

jest rosnąca.

Dowód.

Niech \(\displaystyle{ x_1, x_2 \in \RR_+}\) oraz \(\displaystyle{ x_1<x_2.}\) Wtedy \(\displaystyle{ 0<\frac{x_1}{x_2}<1,}\) zatem na mocy nierówności Bernoulliego

\(\displaystyle{ \left(1+\frac{1}{x_1} \right)^{x_1} = \left(1+\frac{1}{x_1} \right)^{\frac{x_1}{x_2} \cdot x_2} < \left(1+ \frac{x_1}{x_2} \cdot \frac{1}{x_1} \right)^{x_2} = \left(1+\frac{1}{x_2} \right)^{x_2}.}\) \(\displaystyle{ \blacktriangledown}\)

Uwaga! Z powyższego dowodu wynika w szczególności monotoniczność ciągu \(\displaystyle{ \left( e_n \right).}\)


Lemat 3. \(\displaystyle{ \lim_{x \to 0^+} (1+x)^{\frac{1}{x}} = e.}\)

Dowód.

Ustalmy \(\displaystyle{ \varepsilon>0.}\) Skoro

\(\displaystyle{ e=\lim_{n \to \infty} \left(1+\frac{1}{n} \right)^n,}\)

to istnieje takie \(\displaystyle{ N_0 \in \mathbb N,}\) że gdy \(\displaystyle{ n \ge N_0,}\) to

\(\displaystyle{ \left| \left(1+\frac{1}{n} \right)^n -e \right|< \varepsilon.}\)

Weźmy dowolny \(\displaystyle{ x \in \left(0, \frac{1}{N_0} \right).}\) Skoro \(\displaystyle{ \frac{1}{x} > N_0,}\) to

\(\displaystyle{ (1+x)^{\frac{1}{x}}=\left(1+ \frac{1}{\frac{1}{x}} \right)^{\frac{1}{x}} > \left(1+\frac{1}{N_0} \right)^{N_0}>e-\varepsilon,}\)

bo, na mocy faktu 2, funkcja \(\displaystyle{ \left(1+\frac{1}{x} \right)^x}\) jest rosnąca na \(\displaystyle{ \RR_+.}\)
Jednocześnie, istnieje takie \(\displaystyle{ m \in \mathbb N,}\) że \(\displaystyle{ \frac{1}{x}<m,}\) a co za tym idzie

\(\displaystyle{ (1+x)^{\frac{1}{x}} < \left(1+\frac{1}{m} \right)^m < e,}\)

przy czym druga nierówność wynika z faktu, że monotoniczny ciąg zbieżny jest ograniczony przez swoją granicę. Otrzymaliśmy więc, że dla \(\displaystyle{ x \in \left (0, \frac{1}{N_0} \right)}\) jest

\(\displaystyle{ e-\varepsilon < (1+x)^{\frac{1}{x} } < e,}\)

zatem \(\displaystyle{ \lim_{x \to 0^+} (1+x)^{\frac{1}{x}} = e.}\) \(\displaystyle{ \blacktriangledown}\)


Lemat 4. \(\displaystyle{ \lim_{x \to 0^-} (1+x)^{\frac{1}{x}} = e.}\)

Dowód.

Gdy \(\displaystyle{ x \to 0^{-},}\) to \(\displaystyle{ \frac{-x}{1+x} \to 0^+,}\) zatem

\(\displaystyle{ \lim_{x \to 0^-} (1+x)^{\frac{1}{x}} = \lim_{x \to 0^-} \left( \frac{1}{1+x} \right)^{\frac{1}{-x}} = \lim_{x \to 0^-} \left( 1+\frac{-x}{1+x} \right)^{\frac{1+x}{-x} + 1} = \\ \\ = \lim_{x \to 0^-} \left( 1+\frac{-x}{1+x} \right)^{\frac{1+x}{-x}} \cdot \left( 1+\frac{-x}{1+x} \right) = e \cdot \left( 1+\frac{0}{1+0} \right) = e. \ \blacktriangledown}\)


Twierdzenie. \(\displaystyle{ \lim_{x \to 0} (1+x)^{\frac{1}{x}} = e.}\)

Dowód.

Wynika to natychmiast z dwóch poprzednich lematów. \(\displaystyle{ \blacktriangledown}\)


Dopiero w tym momencie możemy korzystać z faktu, że \(\displaystyle{ e= \lim_{n \to \infty} \left(1+a_n \right)^{\frac{1}{a_n}}}\) dla dowolnego ciągu \(\displaystyle{ \left( a_n \right)}\) zbieżnego do zera i o niezerowych wyrazach. Gdybyśmy na początku w miejsce definicji \(\displaystyle{ \mbox{I}}\) przyjęli definicję \(\displaystyle{ \mbox{II},}\) niestety należałoby pokazać, że owa granica istnieje, a to mogłoby być niewiele łatwiejsze od drogi, którą dopiero co przebyliśmy.
Na zakończenie zaprezentuję dwie podobne granice, których sposoby liczenia odrobinę różnią się ze względu na potrzebną wiedzę.


1. \(\displaystyle{ \lim_{n \to \infty} \left(1+\frac{1}{3n} \right)^{3n}=e.}\)

\(\displaystyle{ \left(1+\frac{1}{3n} \right)^{3n} = e_{3n}}\) oraz \(\displaystyle{ \lim_{n \to \infty} e_n = e,}\) więc granica podciągu \(\displaystyle{ \left( e_{3n} \right)}\) musi być taka sama.


2. \(\displaystyle{ \lim_{n \to \infty} \left( 1+\frac{7}{2n} \right)^{\frac{2n}{7}} = e.}\)

Tu niestety nie ma żadnej możliwości skorzystania z ciągu \(\displaystyle{ e_n,}\) więc do uzasadnienia powyższej równości potrzebujemy powołać się na co najmniej fragment przedstawionych w tym artykule faktów i lematów.


Wszelakie uwagi na PW bardzo mile widziane.
ODPOWIEDZ